Storing Cookies (See : http://ec.europa.eu/ipg/basics/legal/cookies/index_en.htm ) help us to bring you our services at overunity.com . If you use this website and our services you declare yourself okay with using cookies .More Infos here:
https://overunity.com/5553/privacy-policy/
If you do not agree with storing cookies, please LEAVE this website now. From the 25th of May 2018, every existing user has to accept the GDPR agreement at first login. If a user is unwilling to accept the GDPR, he should email us and request to erase his account. Many thanks for your understanding

User Menu

Custom Search

Author Topic: Pierre's 170W in 1600W out Looped Very impressive Build continued & moderated  (Read 433018 times)

pmgr

  • Full Member
  • ***
  • Posts: 184
    • Stop organ harvesting from Falun Gong practitioners
Hi Jerdee,

The way the fields are mixed isn't by just simple superposition (i.e. addition) of the fields. To have more energy out than spent to make the field it has to become coherent like done in LASER light. That way the waves don't ride through each other like two beams of light crossing but becomes a new single wave representing the addition before doing the squaring (coherent instead of simple superposition). Normally to make the separate waves become a single coherent wave the single waves has to mixed in a nonlinear media. Else the waves just slip apart or ride through each other which is normal superposition or interference. The question then is: Is the core working around the point of magnetic saturation?

My guess is that Pierre's device actually manages to mix or merge the separate fields of each coil into a new coherent field that doesn't slip apart. The ratios of energy then becomes:

Input energy: 12 + 12 + 12 + 12 + 12 + 12 = 6
Output energy: (1 + 1 + 1 + 1 + 1 + 1)2 = 36

Or actually it's 12 coils (six coils in each end) giving the difference in energy levels of 12 times:
Input energy: 12
Output energy: 122 = 144

This is just my guess. The ratio will have to be measured.

Regards
Ole
Ole, as a laser physicist, I have to disagree with you. Laser light is indeed coherent, but that doesn't mean that the energy contained in it is more than what was needed to create it.

In coherent optical communication, you can mix a local oscillator with an incoming laser beam to amplify it upon detection. A photodiode which detects light has a absolute value squaring function: |E|^2.

So if you put the incoming beam (Ein) directly on the detector, you measure |Ein|^2. If you mix it with a local oscillator first, you get |Ein + Eloc|^2 which gives |Ein|^2 + |Eloc|^2 + 2 Re{Ein*conj(Eloc)}. In a real application, the last term is of interest as its amplitude is magnified by the power of the local laser, but again, the total energy is never larger than the sum of the energy of the incoming laser and the local oscillator.

PmgR

onielsen

  • Newbie
  • *
  • Posts: 28
Hi pmgr,

Quote
Ole, as a laser physicist, I have to disagree with you. Laser light is indeed coherent, but that doesn't mean that the energy contained in it is more than what was needed to create it.
I don't know from where else the excess energy arrives then. Dr. A. Melnichenko discovered by accident this principle when he melted a microwave guide which is a rectangular copper tube. He sent microwaves from two or was it three transmitters through the wave guide. The power wasn't enough to melt the tube which it did anyway. His explanation to the phenomena was that the beams became coherent which could account for the excess power that ruined the wave guide.

Two of Melnichenko's articles explaining the principle:
Generator of energy on nonlinear inductance: https://translate.google.dk/translate?hl=en&sl=ru&tl=en&u=http%3A%2F%2Fua-hho.do.am%2Fpubl%2Fidei_po_ehnergii%2Fgenerator_ehnergii_na_nelinejnoj_induktivnosti%2F2-1-0-185
In-phase interference of electromagnetic waves and energy transgeneration: http://translate.google.com/translate?depth=1&hl=en&ie=UTF8&prev=_t&rurl=translate.google.com&sl=auto&tl=en&twu=1&u=http://izob.narod.ru/p0007.html

János Vajda has a mathematical treatise on the principle:
 VIOLATION OF THE LAW OF ENERGY CONSERVATION IN WAVE FIELDS: https://feprinciples.files.wordpress.com/2016/02/energy_from_wave_fields_1-21.pdf

Try looking at it from an engineer's point of view. If it works even though many books will tell that it doesn't work perhaps the books should be rewritten to fit nature instead of dictating nature to fit the books. Read Ivor Catt's struggle the scientific community to see the difference between scientists  and engineers or technicians. Eventually read about 'The Forbidden Equation: i = qc' to see what is wrong in the scientific community: (https://wiki.naturalphilosophy.org/index.php?title=The_Forbidden_Equation:_i_%3D_qc)

It all comes down to removing the properties of space. Space has permittivity and permeability which are physical properties and can be simply measured. Perhaps there is an aeather (electrons and positrons according to W.B. Smith' theory) even though many books discards it?!

Regards
Ole

gotoluc

  • elite_member
  • Hero Member
  • ******
  • Posts: 3096


6 Coil Test AC vs DC: https://youtu.be/hrMSWq6iZcE

pmgr

  • Full Member
  • ***
  • Posts: 184
    • Stop organ harvesting from Falun Gong practitioners

6 Coil Test AC vs DC: https://youtu.be/hrMSWq6iZcE
Luc, can you post a schematic for how you are driving your coils, as well as the  propellor connections/code. It's hard to figure out how things are connected and biased, just listening to your explanation.
PmgR

konehead

  • Sr. Member
  • ****
  • Posts: 462
Hi Luc
Great video again and great progress too
Numbers now looking real good!
All makes sense and simple now...and why in retrospect hitt the poor rotor winds from all sides

Think of rotor winds as permanent magnet and rotating fields revolve as in rotating generator and then how is best way to do it....
Also more important of course think of rotatng fields as permanent magnets and rotor as it is.....now how could you do the best rotating generator like this?
I would say your best bet with 30 poles in stator is to "mimic" a revolving field of.10 permanent magnets and energize always only two at 180 degrees that are in that "edge" register you found.
Space the revolving fields apart such as only one pole is energized ( actually two at 180) and there  is  blank pole each side
Idea is to mimic a rotating field of 10 permanent magnets 
nsnsnsnsn and the width of permanent magnets is also the same distance as that between them
Soi12mm wide magnets now 12mm ispace in between measuring edge to edge
Now, mimic eletomsonically via Jerdee's expertise this revolving permanent magnet field whirling  around that rotor with the 600 turns
So it becomes  electromagnet whirling field of course.
I base all this advice on the many Muller generators I have built
These with flat disc rotors stuffed with neo magnets evenly spaced in the flat disc then coils each side
Common design 8 or 16 NS magnets in rotor
And 9 or17 coils each side of rotor
But of course you have a single coil to whirl magnets around
Anyways just think if 12 mm wide magnets have 12mm space between them translate this dz generator mode
I think this will work  very good for you
Higher frequencies should make incredible power




jerdee

  • Newbie
  • *
  • Posts: 49
Luc, can you post a schematic for how you are driving your coils, as well as the  propellor connections/code. It's hard to figure out how things are connected and biased, just listening to your explanation.
PmgR

Code might be a bit confusing, as it is all in binary.  So posting the code will most likely be more confusing for most.

We are trying to focus on creating the strongest field on the stator windings that has the least amount of cancellation.

H-Bridges are configured this way all the way around in series.
H-bridge 1 beginning of coil 1 to the end of 5
H-bridge 2 beginning of coil 2 to the end of 6.
etc...

The LOOP in laymen terms is this...

REPEAT
  NORTH is created on coils 2 through 6 while SOUTH is created on coils 17 through 21
  NORTH is created on coils 3 through 7 while SOUTH is created on coils 18 through 22
  NORTH is created on coils 4 through 8 while SOUTH is created on coils 19 through 23
  SOUTH is created on coils 2 through 6 while NORTH is created on coils 17 through 21
  SOUTH is created on coils 3 through 7 while NORTH is created on coils 18 through 22
  SOUTH is created on coils 4 through 8 while NORTH is created on coils 19 through 23

This test is explaining our issues of why we are not getting enough flux across the armature.

We need to isolate the poles.  This test is to eliminate the other poles as much as possible before separating the series coils arrangement.  So only two poles are active at one time in A/C mode.

I don't see how moving forward with 5 coils in series overlapping per pole helps.  I believe the test is proving this.

Hope this helps,
Jerdee

partzman

  • Sr. Member
  • ****
  • Posts: 379

6 Coil Test AC vs DC: https://youtu.be/hrMSWq6iZcE

Luc,

You must have missed my post on OUR on 2018-04-17 regarding this very topology so I will re-post here-

"Much confusion seems to exist on OU with Pierre's device and it's operation regarding the switching of his 36 serially connected stator coils.  The output levels of the builder's attempts thus far are low which indicates there is a flaw in their thinking.  So, in an attempt to resolve the problem, I simulated the power transformer feeding the supercap (SC) pack while supplying the running stator without the rotor inserted.  The idea was to replicate Pierre's SC loaded output voltage with the known input voltage and current to the power transformer under these conditions.  No real magic here but a few parameters had to be assumed such as coupling factor, etc.

Considering the assumed and accepted switching connections by the replicators plus their posted coil dcrs, a relay switched arrangement should present a load in the ~.5-1 ohm range to the SC pack.  When these load numbers were used in the sim, the results were way off from Pierre's measurements and I posted this info on OU to no avail.  IMO, the assumed switch connections are wrong and some basics are being overlooked that is, the voltage and current waveform generated by passing a PM over a coil.  So, this post represents my thoughts on the matter.

Attached is an 18 slot sim for simplification to get the point across.  The current measurements for each coil used are offset for clarity.  It consists of 2 poles that are physically and electrically opposite from each other namely L1,2,3 and L10,11,12.  All other connected coils have zero current and the reason why is apparent if the circuit is studied.  This is the key because if we now do average measurements on the voltages and currents, we arrive at a net load resistance in the range that allow the original power transformer sim to work.  IOW, not all coils are energized all the time.  The switching to move the fields around the stator should be academic from here based on the number of poles desired and the slot pitch.

Any recovery from the inductive switching can now be visualized from the moving electromagnetic field over the stationary loaded rotor similar to a PM passing over coil."

The referenced sim is attached.

Regards,
Pm

pmgr

  • Full Member
  • ***
  • Posts: 184
    • Stop organ harvesting from Falun Gong practitioners
Code might be a bit confusing, as it is all in binary.  So posting the code will most likely be more confusing for most.

We are trying to focus on creating the strongest field on the stator windings that has the least amount of cancellation.

H-Bridges are configured this way all the way around in series.
H-bridge 1 beginning of coil 1 to the end of 5
H-bridge 2 beginning of coil 2 to the end of 6.
etc...

The LOOP in laymen terms is this...

REPEAT
  NORTH is created on coils 2 through 6 while SOUTH is created on coils 17 through 21
  NORTH is created on coils 3 through 7 while SOUTH is created on coils 18 through 22
  NORTH is created on coils 4 through 8 while SOUTH is created on coils 19 through 23
  SOUTH is created on coils 2 through 6 while NORTH is created on coils 17 through 21
  SOUTH is created on coils 3 through 7 while NORTH is created on coils 18 through 22
  SOUTH is created on coils 4 through 8 while NORTH is created on coils 19 through 23

This test is explaining our issues of why we are not getting enough flux across the armature.

We need to isolate the poles.  This test is to eliminate the other poles as much as possible before separating the series coils arrangement.  So only two poles are active at one time in A/C mode.

I don't see how moving forward with 5 coils in series overlapping per pole helps.  I believe the test is proving this.

Hope this helps,
Jerdee
Luc/Jerdee,


What timings are you using and overlap time? This is important as it will effect current draw.


Also, for this configuration, turn up the frequency and see what you get. I predict that the output coil voltage won't increase much more once you go past the stator/rotor max frequency, which is around 50-60Hz. Please see if you can confirms this.


If so, it simply means your rotor is too thick and was collapsing poles in your earlier measurements and your frequency was too high to see any effect with the thinner rotor.


Also repeat your current measurement with the thinner rotor at low frequency and at high frequency. Then you will now what frequency you need to aim for and if you need to make another rotor with an intermediate thickness.


Lastly, see what happens if you extend your single North and South poles in your code above to span half of your rotor, so bias coils 1-15 North, 16-30 South and rotate around.


PmgR

jerdee

  • Newbie
  • *
  • Posts: 49
Quote
What timings are you using and overlap time? This is important as it will effect current draw.

I've found it the easiest to use two "wait" or "pause" commands.  So we can always adjust overlap and hold times.  You are correct, the more overlap, the more demand.  Not good. We've found that most of the overlap is not needed.  It's waisted. 

Quote
Also, for this configuration, turn up the frequency and see what you get. I predict that the output coil voltage won't increase much more once you go past the stator/rotor max frequency, which is around 50-60Hz. Please see if you can confirms this.

Higher freq...provides less current, so you have to input more voltage.  Nothing unusual.

Quote
Also repeat your current measurement with the thinner rotor at low frequency and at high frequency. Then you will now what frequency you need to aim for and if you need to make another rotor with an intermediate thickness.

Our mains focus at this moment is getting as much of the rotating magnetic field as possible.  We are both in agreement that isolation of the pole fields are needed.

Quote
Lastly, see what happens if you extend your single North and South poles in your code above to span half of your rotor, so bias coils 1-15 North, 16-30 South and rotate around.

Yes, this makes sense.  This requires the slot arrangment to be reconfigured.  Not 5 slots, but 15 slots.  Also this currently requires 15 times 3 pins from MCU.  Sadly we can't do this yet.  Our gates came in today.  So we have more pinout control from MCU.  Otherwise we run out quickly in pin control from MCU.  I do not think we need to go to higher pin out from MCU to get this device to work.  We are going to be using XOR gates to drop our pin control down by one third. You can run multipled poles in parallel, and save pin count as well.

Isolation of the pole fields is key.   If you can't get the flux to "re-connect" in the armature...you have little results.  This is how I see it at this point.

Jerdee

gotoluc

  • elite_member
  • Hero Member
  • ******
  • Posts: 3096
Luc,

You must have missed my post on OUR on 2018-04-17 regarding this very topology so I will re-post here-

"Much confusion seems to exist on OU with Pierre's device and it's operation regarding the switching of his 36 serially connected stator coils.  The output levels of the builder's attempts thus far are low which indicates there is a flaw in their thinking.  So, in an attempt to resolve the problem, I simulated the power transformer feeding the supercap (SC) pack while supplying the running stator without the rotor inserted.  The idea was to replicate Pierre's SC loaded output voltage with the known input voltage and current to the power transformer under these conditions.  No real magic here but a few parameters had to be assumed such as coupling factor, etc.

Considering the assumed and accepted switching connections by the replicators plus their posted coil dcrs, a relay switched arrangement should present a load in the ~.5-1 ohm range to the SC pack.  When these load numbers were used in the sim, the results were way off from Pierre's measurements and I posted this info on OU to no avail.  IMO, the assumed switch connections are wrong and some basics are being overlooked that is, the voltage and current waveform generated by passing a PM over a coil.  So, this post represents my thoughts on the matter.

Attached is an 18 slot sim for simplification to get the point across.  The current measurements for each coil used are offset for clarity.  It consists of 2 poles that are physically and electrically opposite from each other namely L1,2,3 and L10,11,12.  All other connected coils have zero current and the reason why is apparent if the circuit is studied.  This is the key because if we now do average measurements on the voltages and currents, we arrive at a net load resistance in the range that allow the original power transformer sim to work.  IOW, not all coils are energized all the time.  The switching to move the fields around the stator should be academic from here based on the number of poles desired and the slot pitch.

Any recovery from the inductive switching can now be visualized from the moving electromagnetic field over the stationary loaded rotor similar to a PM passing over coil."

The referenced sim is attached.

Regards,
Pm


Thanks for sharing your simulation Partzman

I have not been following the topic at Over Unity Research (OUR)   kind of busy with just this topic here.

If anyone is interested, here is the link to the first page:  http://www.overunityresearch.com/index.php?topic=3599.0
Kind regards
Luc

pmgr

  • Full Member
  • ***
  • Posts: 184
    • Stop organ harvesting from Falun Gong practitioners
I've found it the easiest to use two "wait" or "pause" commands.  So we can always adjust overlap and hold times.  You are correct, the more overlap, the more demand.  Not good. We've found that most of the overlap is not needed.  It's waisted. 

Higher freq...provides less current, so you have to input more voltage.  Nothing unusual.
This is what I have been saying all along. The iron will cut the frequency off. So everything is following normal EM theory.


Quote

Our mains focus at this moment is getting as much of the rotating magnetic field as possible.  We are both in agreement that isolation of the pole fields are needed.
This is incorrect. Pole isolation is not required as long as you drive all poles with the same current. Your experiments are further proof that your thick stator is shorting poles and thus reducing the output coil voltage. Your thin rotor is most likely too thin and its reluctance is too high, also limiting output voltage. So go for a rotor thickness somewhere in between and use a laminated rotor.

Quote

Yes, this makes sense.  This requires the slot arrangment to be reconfigured.  Not 5 slots, but 15 slots.  Also this currently requires 15 times 3 pins from MCU.  Sadly we can't do this yet.  Our gates came in today.  So we have more pinout control from MCU.  Otherwise we run out quickly in pin control from MCU.  I do not think we need to go to higher pin out from MCU to get this device to work.  We are going to be using XOR gates to drop our pin control down by one third. You can run multipled poles in parallel, and save pin count as well.

You don't need to change anything on the stator. Leave the coilspan as 5 slots as you currently have.
How many pins do you have on your MCU? You should be able to do this with your current arrangement with 30pins only (two bridges are always on; one high side and one low side). For each bridge, simply hook up the high side switch pin to +5V, the low switch pin to GND and connect the enable pins to the Arduino.

Quote
Isolation of the pole fields is key.   If you can't get the flux to "re-connect" in the armature...you have little results.  This is how I see it at this point.
Jerdee
I disagree. If you properly bias all poles with equal current, no pole isolation is required. I have simulated this in FEMM and there is no problem if the pole currents are all equal and properly phased as I have described before. However, with the original schematic Pierre that showed, this is not possible.


PmgR
« Last Edit: June 08, 2018, 07:42:48 AM by pmgr »

AlienGrey

  • Hero Member
  • *****
  • Posts: 3713
Err will you be doing a hardware kit for this device or where to get the parts in your location ???   :D :D :D :D :D :D

gotoluc

  • elite_member
  • Hero Member
  • ******
  • Posts: 3096
36 bobine peuvent être relier avec une autre et vous pouvez isoler chaque bobine avec un relais

En. 36 coils can be connected with another and you can isolate each coil with a relay

Une question pour toi Pierre.
Les 36 bobines de ton DZ fonctionnel, était-elle individuellement isolées?

Cordialement
Luc

En. A question for you Pierre.
The 36 coils of your functional DZ,  were they individually isolated?

Regards
Luc

seaad

  • Sr. Member
  • ****
  • Posts: 311
Une question pour toi Pierre.
Les 36 bobines de ton DZ fonctionnel, était-elle individuellement isolées?

En. A question for you Pierre.
The 36 coils of your functional DZ,  were they individually isolated?

Regards
Luc

Maybe to few wires (36, + some more?) from the stator to manage that?

 --  Why then make/ show the fifth DZ film?
 --  And this? https://overunity.com/17653/pierres-170w-in-1600w-out-looped-very-impressive-build-continued-moderated/msg522229/#msg522229

Regards Arne

jerdee

  • Newbie
  • *
  • Posts: 49

First, thanks for your help PmgR

Quote
How many pins do you have on your MCU? You should be able to do this with your current arrangement with 30pins only (two bridges are always on; one high side and one low side). For each bridge, simply hook up the high side switch pin to +5V, the low switch pin to GND and connect the enable pins to the Arduino.

Yes, we are thinking in the same direction .  We are doing this moving forward. We still need the logic chips to save pin count considerably.

Quote
if the pole currents are all equal and properly phased


Yes, without a doubt.   

This development requires more indivdual bridge control.

Again, thanks for your help.

Jerdee